www.vorhilfe.de
- Förderverein -
Der Förderverein.

Gemeinnütziger Verein zur Finanzierung des Projekts Vorhilfe.de.
Hallo Gast!einloggen | registrieren ]
Startseite · Mitglieder · Impressum
Forenbaum
^ Forenbaum
Status VH e.V.
  Status Vereinsforum

Gezeigt werden alle Foren bis zur Tiefe 2

Navigation
 Startseite...
 Suchen
 Impressum
Das Projekt
Server und Internetanbindung werden durch Spenden finanziert.
Organisiert wird das Projekt von unserem Koordinatorenteam.
Hunderte Mitglieder helfen ehrenamtlich in unseren moderierten Foren.
Anbieter der Seite ist der gemeinnützige Verein "Vorhilfe.de e.V.".
Partnerseiten
Weitere Fächer:

Open Source FunktionenplotterFunkyPlot: Kostenloser und quelloffener Funktionenplotter für Linux und andere Betriebssysteme
Forum "Wahrscheinlichkeitstheorie" - Verteilungskonvergenz
Verteilungskonvergenz < Wahrscheinlichkeitstheorie < Stochastik < Hochschule < Mathe < Vorhilfe
Ansicht: [ geschachtelt ] | ^ Forum "Wahrscheinlichkeitstheorie"  | ^^ Alle Foren  | ^ Forenbaum  | Materialien

Verteilungskonvergenz: Frage (überfällig)
Status: (Frage) überfällig Status 
Datum: 23:49 Mi 27.09.2006
Autor: Wolff

Beweisidee:
Aufgabe
Seien [mm] $X_{n}, [/mm] n [mm] \ge [/mm] 0$ nicht-negative ZV mit [mm] $X_{n}\to\infty$ [/mm] in Verteilung und [mm] $Z_{n}, [/mm] n [mm] \ge [/mm] 0$ ZV mit [mm] $Z_{n}:=I\{A_{n}\}$ [/mm] mit [mm] $A_{n}\in\mathcal{A}$. [/mm]
Dann ist  
[mm] $\limes_{n\rightarrow\infty}EZ_{n}=0 \gdw \limes_{n\rightarrow\infty}\bruch{E[e^{-X_{n}}*Z_{n}]}{E[e^{-X_{n}}]}=0$. [/mm]



Ich habe diese Frage in keinem Forum auf anderen Internetseiten gestellt.


        
Bezug
Verteilungskonvergenz: Mitteilung
Status: (Mitteilung) Reaktion unnötig Status 
Datum: 00:07 Do 28.09.2006
Autor: felixf

Hallo!

> Beweisidee:
>  Seien [mm]X_{n},[/mm] n [mm]\ge[/mm] 0 nicht-negative ZV mit [mm]X_{n}\to\infty[/mm]
> in Verteilung
>  und [mm]Z_{n},[/mm] n [mm]\ge[/mm] 0 ZV mit [mm]Z_{n}:=I\{A_{n}\}[/mm] mit
> [mm]A_{n}\in\mathcal{A}.[/mm]
>  Dann ist  
> [mm]\limes_{n\rightarrow\infty}EZ_{n}=0 \gdw \limes_{n\rightarrow\infty}\bruch{E[e^{-X_{n}}*Z_{n}]}{E[e^{-X_{n}}]}=0.[/mm]

Was ist hier die Frage? Soll das die Idee fuer einen Beweis von einer Aussage sein (wenn ja, von welcher)? Oder willst du eine Beweisidee fuer die [mm] $\gdw$-Aussage [/mm] hier? Lies dir doch bitte mal die Foren-Regeln durch und beherzige sie ein wenig :-)

LG Felix


Bezug
                
Bezug
Verteilungskonvergenz: Mitteilung
Status: (Mitteilung) Reaktion unnötig Status 
Datum: 08:49 Do 28.09.2006
Autor: Wolff

Beweisidee fuer die [mm] \gdw-Aussage [/mm] hier.
Ich bin nicht sicher, ob beide Richtungen bewiesen werden können. Wenn nicht, können Sie Gegenbeispiel geben?
Danke für Ihre Hilfe!

Bezug
        
Bezug
Verteilungskonvergenz: Fälligkeit abgelaufen
Status: (Mitteilung) Reaktion unnötig Status 
Datum: 00:20 So 01.10.2006
Autor: matux

$MATUXTEXT(ueberfaellige_frage)
Bezug
Ansicht: [ geschachtelt ] | ^ Forum "Wahrscheinlichkeitstheorie"  | ^^ Alle Foren  | ^ Forenbaum  | Materialien


^ Seitenanfang ^
ev.vorhilfe.de
[ Startseite | Mitglieder | Impressum ]